Identity with repeatedly taking the commutator of a ring element












3














This is taken from Jacobson's Basic Algebra 2e, it's 2.1.5



If $a$ and $b$ are elements of a ring, define $a^{(0)} =a, a^{(1)} = [a,b] = ab-ba$ and $a^{(k)}=[a^{(k-1)},b]$ Prove the following formula: $$sum_{i=0}^k b^i a b^{k-i} = sum_{j=0}^k {k+1 choose j+1} b^{k-j}a^{(j)}$$



So, I want to use induction to prove this and have verified it for k=1 and k=2. I've worked on the left hand side and gotten $$sum_{i=0}^k b^i a b^{k-i} =(sum_{i=0}^{k-1} b^i a b^{(k-1)-i} )b + b^k a $$
The next part would be to make $$ sum_{j=0}^{k-1} {k choose j} b^{(k-1)-j}a^{(j)}$$ appear on the right hand side so that I can apply the inductive hypothesis. The only thing I can think of would be to use ${n+1 choose k}={n choose k } + {n choose k-1}$. Maybe I've made an error but I believe this gives $$sum_{j=0}^k {k+1 choose j+1} b^{k-j}a^{(j)}=b(sum_{j=0}^{k-1} {k choose j+1} b^{(k-1)-j}a^{(j)}) + sum_{j=0}^k {k choose j}b^{k-j}a^{(j)} + {k choose k+1}b^0 a^{(k)}$$
While the last term is $0$, because the extra $b$ appears on opposite sides of the first term, I can't easily equate them and cancel. So I think I'm barking up the wrong tree trying to manipulate the right hand side of the formula in this way.



My question is two fold: how to prove this identity, and what does this composition of the commutator $a^{(j)}$ represent? If it eventually hits 0 is that still some kind of measure for how near $a$ and $b$ are to commuting? If anybody has seen this identity before and it has some usefulness beyond the exercise of proving it, I would also love to hear that.










share|cite|improve this question






















  • I haven't properly thought about your approach yet, but here is the "standard" trick for this sort of identity: Let $A$ be the ring. Let $L : A to A$ be the map sending each $x$ to $bx$, and let $R : A to A$ be the map sending each $x$ to $xb$. Then, the operators $L$ and $R$ are $mathbb{Z}$-linear and commute. But the left hand side of your identity is $sumlimits_{i=0}^k L^i R^{k-i} a$, whereas the right hand side is $sumlimits_{j=0}^k dbinom{k+1}{j+1} L^{k-j} left(R-Lright)^j a$. So it remains to ...
    – darij grinberg
    Nov 20 '18 at 6:19










  • ... prove that $sumlimits_{i=0}^k L^i R^{k-i} = sumlimits_{j=0}^k dbinom{k+1}{j+1} L^{k-j} left(R-Lright)^j$. This should follow from binomial-style manipulations (treating $L$ and $R$ as two arbitrary commuting elements).
    – darij grinberg
    Nov 20 '18 at 6:19












  • Ah, yes, the identity $sumlimits_{i=0}^k x^i y^{k-i} = sumlimits_{j=0}^k dbinom{k+1}{j+1} x^{k-j} left(y-xright)^j$ holds for two arbitrary commuting elements $x$ and $y$. To prove it, it suffices to do so when $x$ and $y$ are two commuting indeterminates in a polynomial ring. Multiply both sides by $x-y$ (this is allowed, since $x-y$ is not a zero-divisor in a polynomial ring), so that the left hand side simplifies to $x^{k+1} - y^{k+1}$. Rewrite this using the binomial formula for $y^{k+1} = left(left(y-xright) + xright)^{k+1}$.
    – darij grinberg
    Nov 20 '18 at 6:22










  • As to your induction... You want to simplify $left(sum_{j=0}^{k-1} dbinom{k}{j} b^{left(k-1right)-j} a^{(j)} right) b$ so that it looks more like $sum_{j=0}^{k} dbinom{k+1}{j} b^{k-j} a^{(j)}$. So you want to commute the $b$ past the $a^{(j)}$. Of course, it doesn't just commute, but you have $a^{(j)} b = a^{(j+1)} + b a^{(j)}$. So your sum splits into two, with one sum getting its index shifted. I think you can finish it from here.
    – darij grinberg
    Nov 20 '18 at 6:28
















3














This is taken from Jacobson's Basic Algebra 2e, it's 2.1.5



If $a$ and $b$ are elements of a ring, define $a^{(0)} =a, a^{(1)} = [a,b] = ab-ba$ and $a^{(k)}=[a^{(k-1)},b]$ Prove the following formula: $$sum_{i=0}^k b^i a b^{k-i} = sum_{j=0}^k {k+1 choose j+1} b^{k-j}a^{(j)}$$



So, I want to use induction to prove this and have verified it for k=1 and k=2. I've worked on the left hand side and gotten $$sum_{i=0}^k b^i a b^{k-i} =(sum_{i=0}^{k-1} b^i a b^{(k-1)-i} )b + b^k a $$
The next part would be to make $$ sum_{j=0}^{k-1} {k choose j} b^{(k-1)-j}a^{(j)}$$ appear on the right hand side so that I can apply the inductive hypothesis. The only thing I can think of would be to use ${n+1 choose k}={n choose k } + {n choose k-1}$. Maybe I've made an error but I believe this gives $$sum_{j=0}^k {k+1 choose j+1} b^{k-j}a^{(j)}=b(sum_{j=0}^{k-1} {k choose j+1} b^{(k-1)-j}a^{(j)}) + sum_{j=0}^k {k choose j}b^{k-j}a^{(j)} + {k choose k+1}b^0 a^{(k)}$$
While the last term is $0$, because the extra $b$ appears on opposite sides of the first term, I can't easily equate them and cancel. So I think I'm barking up the wrong tree trying to manipulate the right hand side of the formula in this way.



My question is two fold: how to prove this identity, and what does this composition of the commutator $a^{(j)}$ represent? If it eventually hits 0 is that still some kind of measure for how near $a$ and $b$ are to commuting? If anybody has seen this identity before and it has some usefulness beyond the exercise of proving it, I would also love to hear that.










share|cite|improve this question






















  • I haven't properly thought about your approach yet, but here is the "standard" trick for this sort of identity: Let $A$ be the ring. Let $L : A to A$ be the map sending each $x$ to $bx$, and let $R : A to A$ be the map sending each $x$ to $xb$. Then, the operators $L$ and $R$ are $mathbb{Z}$-linear and commute. But the left hand side of your identity is $sumlimits_{i=0}^k L^i R^{k-i} a$, whereas the right hand side is $sumlimits_{j=0}^k dbinom{k+1}{j+1} L^{k-j} left(R-Lright)^j a$. So it remains to ...
    – darij grinberg
    Nov 20 '18 at 6:19










  • ... prove that $sumlimits_{i=0}^k L^i R^{k-i} = sumlimits_{j=0}^k dbinom{k+1}{j+1} L^{k-j} left(R-Lright)^j$. This should follow from binomial-style manipulations (treating $L$ and $R$ as two arbitrary commuting elements).
    – darij grinberg
    Nov 20 '18 at 6:19












  • Ah, yes, the identity $sumlimits_{i=0}^k x^i y^{k-i} = sumlimits_{j=0}^k dbinom{k+1}{j+1} x^{k-j} left(y-xright)^j$ holds for two arbitrary commuting elements $x$ and $y$. To prove it, it suffices to do so when $x$ and $y$ are two commuting indeterminates in a polynomial ring. Multiply both sides by $x-y$ (this is allowed, since $x-y$ is not a zero-divisor in a polynomial ring), so that the left hand side simplifies to $x^{k+1} - y^{k+1}$. Rewrite this using the binomial formula for $y^{k+1} = left(left(y-xright) + xright)^{k+1}$.
    – darij grinberg
    Nov 20 '18 at 6:22










  • As to your induction... You want to simplify $left(sum_{j=0}^{k-1} dbinom{k}{j} b^{left(k-1right)-j} a^{(j)} right) b$ so that it looks more like $sum_{j=0}^{k} dbinom{k+1}{j} b^{k-j} a^{(j)}$. So you want to commute the $b$ past the $a^{(j)}$. Of course, it doesn't just commute, but you have $a^{(j)} b = a^{(j+1)} + b a^{(j)}$. So your sum splits into two, with one sum getting its index shifted. I think you can finish it from here.
    – darij grinberg
    Nov 20 '18 at 6:28














3












3








3







This is taken from Jacobson's Basic Algebra 2e, it's 2.1.5



If $a$ and $b$ are elements of a ring, define $a^{(0)} =a, a^{(1)} = [a,b] = ab-ba$ and $a^{(k)}=[a^{(k-1)},b]$ Prove the following formula: $$sum_{i=0}^k b^i a b^{k-i} = sum_{j=0}^k {k+1 choose j+1} b^{k-j}a^{(j)}$$



So, I want to use induction to prove this and have verified it for k=1 and k=2. I've worked on the left hand side and gotten $$sum_{i=0}^k b^i a b^{k-i} =(sum_{i=0}^{k-1} b^i a b^{(k-1)-i} )b + b^k a $$
The next part would be to make $$ sum_{j=0}^{k-1} {k choose j} b^{(k-1)-j}a^{(j)}$$ appear on the right hand side so that I can apply the inductive hypothesis. The only thing I can think of would be to use ${n+1 choose k}={n choose k } + {n choose k-1}$. Maybe I've made an error but I believe this gives $$sum_{j=0}^k {k+1 choose j+1} b^{k-j}a^{(j)}=b(sum_{j=0}^{k-1} {k choose j+1} b^{(k-1)-j}a^{(j)}) + sum_{j=0}^k {k choose j}b^{k-j}a^{(j)} + {k choose k+1}b^0 a^{(k)}$$
While the last term is $0$, because the extra $b$ appears on opposite sides of the first term, I can't easily equate them and cancel. So I think I'm barking up the wrong tree trying to manipulate the right hand side of the formula in this way.



My question is two fold: how to prove this identity, and what does this composition of the commutator $a^{(j)}$ represent? If it eventually hits 0 is that still some kind of measure for how near $a$ and $b$ are to commuting? If anybody has seen this identity before and it has some usefulness beyond the exercise of proving it, I would also love to hear that.










share|cite|improve this question













This is taken from Jacobson's Basic Algebra 2e, it's 2.1.5



If $a$ and $b$ are elements of a ring, define $a^{(0)} =a, a^{(1)} = [a,b] = ab-ba$ and $a^{(k)}=[a^{(k-1)},b]$ Prove the following formula: $$sum_{i=0}^k b^i a b^{k-i} = sum_{j=0}^k {k+1 choose j+1} b^{k-j}a^{(j)}$$



So, I want to use induction to prove this and have verified it for k=1 and k=2. I've worked on the left hand side and gotten $$sum_{i=0}^k b^i a b^{k-i} =(sum_{i=0}^{k-1} b^i a b^{(k-1)-i} )b + b^k a $$
The next part would be to make $$ sum_{j=0}^{k-1} {k choose j} b^{(k-1)-j}a^{(j)}$$ appear on the right hand side so that I can apply the inductive hypothesis. The only thing I can think of would be to use ${n+1 choose k}={n choose k } + {n choose k-1}$. Maybe I've made an error but I believe this gives $$sum_{j=0}^k {k+1 choose j+1} b^{k-j}a^{(j)}=b(sum_{j=0}^{k-1} {k choose j+1} b^{(k-1)-j}a^{(j)}) + sum_{j=0}^k {k choose j}b^{k-j}a^{(j)} + {k choose k+1}b^0 a^{(k)}$$
While the last term is $0$, because the extra $b$ appears on opposite sides of the first term, I can't easily equate them and cancel. So I think I'm barking up the wrong tree trying to manipulate the right hand side of the formula in this way.



My question is two fold: how to prove this identity, and what does this composition of the commutator $a^{(j)}$ represent? If it eventually hits 0 is that still some kind of measure for how near $a$ and $b$ are to commuting? If anybody has seen this identity before and it has some usefulness beyond the exercise of proving it, I would also love to hear that.







ring-theory noncommutative-algebra






share|cite|improve this question













share|cite|improve this question











share|cite|improve this question




share|cite|improve this question










asked Nov 20 '18 at 6:07









MKeller

455




455












  • I haven't properly thought about your approach yet, but here is the "standard" trick for this sort of identity: Let $A$ be the ring. Let $L : A to A$ be the map sending each $x$ to $bx$, and let $R : A to A$ be the map sending each $x$ to $xb$. Then, the operators $L$ and $R$ are $mathbb{Z}$-linear and commute. But the left hand side of your identity is $sumlimits_{i=0}^k L^i R^{k-i} a$, whereas the right hand side is $sumlimits_{j=0}^k dbinom{k+1}{j+1} L^{k-j} left(R-Lright)^j a$. So it remains to ...
    – darij grinberg
    Nov 20 '18 at 6:19










  • ... prove that $sumlimits_{i=0}^k L^i R^{k-i} = sumlimits_{j=0}^k dbinom{k+1}{j+1} L^{k-j} left(R-Lright)^j$. This should follow from binomial-style manipulations (treating $L$ and $R$ as two arbitrary commuting elements).
    – darij grinberg
    Nov 20 '18 at 6:19












  • Ah, yes, the identity $sumlimits_{i=0}^k x^i y^{k-i} = sumlimits_{j=0}^k dbinom{k+1}{j+1} x^{k-j} left(y-xright)^j$ holds for two arbitrary commuting elements $x$ and $y$. To prove it, it suffices to do so when $x$ and $y$ are two commuting indeterminates in a polynomial ring. Multiply both sides by $x-y$ (this is allowed, since $x-y$ is not a zero-divisor in a polynomial ring), so that the left hand side simplifies to $x^{k+1} - y^{k+1}$. Rewrite this using the binomial formula for $y^{k+1} = left(left(y-xright) + xright)^{k+1}$.
    – darij grinberg
    Nov 20 '18 at 6:22










  • As to your induction... You want to simplify $left(sum_{j=0}^{k-1} dbinom{k}{j} b^{left(k-1right)-j} a^{(j)} right) b$ so that it looks more like $sum_{j=0}^{k} dbinom{k+1}{j} b^{k-j} a^{(j)}$. So you want to commute the $b$ past the $a^{(j)}$. Of course, it doesn't just commute, but you have $a^{(j)} b = a^{(j+1)} + b a^{(j)}$. So your sum splits into two, with one sum getting its index shifted. I think you can finish it from here.
    – darij grinberg
    Nov 20 '18 at 6:28


















  • I haven't properly thought about your approach yet, but here is the "standard" trick for this sort of identity: Let $A$ be the ring. Let $L : A to A$ be the map sending each $x$ to $bx$, and let $R : A to A$ be the map sending each $x$ to $xb$. Then, the operators $L$ and $R$ are $mathbb{Z}$-linear and commute. But the left hand side of your identity is $sumlimits_{i=0}^k L^i R^{k-i} a$, whereas the right hand side is $sumlimits_{j=0}^k dbinom{k+1}{j+1} L^{k-j} left(R-Lright)^j a$. So it remains to ...
    – darij grinberg
    Nov 20 '18 at 6:19










  • ... prove that $sumlimits_{i=0}^k L^i R^{k-i} = sumlimits_{j=0}^k dbinom{k+1}{j+1} L^{k-j} left(R-Lright)^j$. This should follow from binomial-style manipulations (treating $L$ and $R$ as two arbitrary commuting elements).
    – darij grinberg
    Nov 20 '18 at 6:19












  • Ah, yes, the identity $sumlimits_{i=0}^k x^i y^{k-i} = sumlimits_{j=0}^k dbinom{k+1}{j+1} x^{k-j} left(y-xright)^j$ holds for two arbitrary commuting elements $x$ and $y$. To prove it, it suffices to do so when $x$ and $y$ are two commuting indeterminates in a polynomial ring. Multiply both sides by $x-y$ (this is allowed, since $x-y$ is not a zero-divisor in a polynomial ring), so that the left hand side simplifies to $x^{k+1} - y^{k+1}$. Rewrite this using the binomial formula for $y^{k+1} = left(left(y-xright) + xright)^{k+1}$.
    – darij grinberg
    Nov 20 '18 at 6:22










  • As to your induction... You want to simplify $left(sum_{j=0}^{k-1} dbinom{k}{j} b^{left(k-1right)-j} a^{(j)} right) b$ so that it looks more like $sum_{j=0}^{k} dbinom{k+1}{j} b^{k-j} a^{(j)}$. So you want to commute the $b$ past the $a^{(j)}$. Of course, it doesn't just commute, but you have $a^{(j)} b = a^{(j+1)} + b a^{(j)}$. So your sum splits into two, with one sum getting its index shifted. I think you can finish it from here.
    – darij grinberg
    Nov 20 '18 at 6:28
















I haven't properly thought about your approach yet, but here is the "standard" trick for this sort of identity: Let $A$ be the ring. Let $L : A to A$ be the map sending each $x$ to $bx$, and let $R : A to A$ be the map sending each $x$ to $xb$. Then, the operators $L$ and $R$ are $mathbb{Z}$-linear and commute. But the left hand side of your identity is $sumlimits_{i=0}^k L^i R^{k-i} a$, whereas the right hand side is $sumlimits_{j=0}^k dbinom{k+1}{j+1} L^{k-j} left(R-Lright)^j a$. So it remains to ...
– darij grinberg
Nov 20 '18 at 6:19




I haven't properly thought about your approach yet, but here is the "standard" trick for this sort of identity: Let $A$ be the ring. Let $L : A to A$ be the map sending each $x$ to $bx$, and let $R : A to A$ be the map sending each $x$ to $xb$. Then, the operators $L$ and $R$ are $mathbb{Z}$-linear and commute. But the left hand side of your identity is $sumlimits_{i=0}^k L^i R^{k-i} a$, whereas the right hand side is $sumlimits_{j=0}^k dbinom{k+1}{j+1} L^{k-j} left(R-Lright)^j a$. So it remains to ...
– darij grinberg
Nov 20 '18 at 6:19












... prove that $sumlimits_{i=0}^k L^i R^{k-i} = sumlimits_{j=0}^k dbinom{k+1}{j+1} L^{k-j} left(R-Lright)^j$. This should follow from binomial-style manipulations (treating $L$ and $R$ as two arbitrary commuting elements).
– darij grinberg
Nov 20 '18 at 6:19






... prove that $sumlimits_{i=0}^k L^i R^{k-i} = sumlimits_{j=0}^k dbinom{k+1}{j+1} L^{k-j} left(R-Lright)^j$. This should follow from binomial-style manipulations (treating $L$ and $R$ as two arbitrary commuting elements).
– darij grinberg
Nov 20 '18 at 6:19














Ah, yes, the identity $sumlimits_{i=0}^k x^i y^{k-i} = sumlimits_{j=0}^k dbinom{k+1}{j+1} x^{k-j} left(y-xright)^j$ holds for two arbitrary commuting elements $x$ and $y$. To prove it, it suffices to do so when $x$ and $y$ are two commuting indeterminates in a polynomial ring. Multiply both sides by $x-y$ (this is allowed, since $x-y$ is not a zero-divisor in a polynomial ring), so that the left hand side simplifies to $x^{k+1} - y^{k+1}$. Rewrite this using the binomial formula for $y^{k+1} = left(left(y-xright) + xright)^{k+1}$.
– darij grinberg
Nov 20 '18 at 6:22




Ah, yes, the identity $sumlimits_{i=0}^k x^i y^{k-i} = sumlimits_{j=0}^k dbinom{k+1}{j+1} x^{k-j} left(y-xright)^j$ holds for two arbitrary commuting elements $x$ and $y$. To prove it, it suffices to do so when $x$ and $y$ are two commuting indeterminates in a polynomial ring. Multiply both sides by $x-y$ (this is allowed, since $x-y$ is not a zero-divisor in a polynomial ring), so that the left hand side simplifies to $x^{k+1} - y^{k+1}$. Rewrite this using the binomial formula for $y^{k+1} = left(left(y-xright) + xright)^{k+1}$.
– darij grinberg
Nov 20 '18 at 6:22












As to your induction... You want to simplify $left(sum_{j=0}^{k-1} dbinom{k}{j} b^{left(k-1right)-j} a^{(j)} right) b$ so that it looks more like $sum_{j=0}^{k} dbinom{k+1}{j} b^{k-j} a^{(j)}$. So you want to commute the $b$ past the $a^{(j)}$. Of course, it doesn't just commute, but you have $a^{(j)} b = a^{(j+1)} + b a^{(j)}$. So your sum splits into two, with one sum getting its index shifted. I think you can finish it from here.
– darij grinberg
Nov 20 '18 at 6:28




As to your induction... You want to simplify $left(sum_{j=0}^{k-1} dbinom{k}{j} b^{left(k-1right)-j} a^{(j)} right) b$ so that it looks more like $sum_{j=0}^{k} dbinom{k+1}{j} b^{k-j} a^{(j)}$. So you want to commute the $b$ past the $a^{(j)}$. Of course, it doesn't just commute, but you have $a^{(j)} b = a^{(j+1)} + b a^{(j)}$. So your sum splits into two, with one sum getting its index shifted. I think you can finish it from here.
– darij grinberg
Nov 20 '18 at 6:28










1 Answer
1






active

oldest

votes


















1














I outlined two solutions in the comments above; let me expand one of them (the
inductive one) into full detail in order to have this question answered. Be
warned: This is going to be a long computation with no twists or surprises.




Theorem 1. Let $a$ and $b$ be two elements of an (associative, unital,
noncommutative) ring $R$. For any $xin R$ and $yin R$, we define the
commutator $left[ x,yright] in R$ of $x$ and $y$ by $left[
x,yright] =xy-yx$
. Define a sequence $left( a^{left( 0right)
},a^{left( 1right) },a^{left( 2right) },ldotsright) $
of elements
of $R$ recursively by setting
begin{align*}
a^{left( 0right) } & =aqquadtext{and}\
a^{left( kright) } & =left[ a^{left( k-1right) },bright]
qquadtext{for each }kgeq 1.
end{align*}

Then,
begin{equation}
sum_{i=0}^{k} b^i ab^{k-i}
= sum_{j=0}^{k}dbinom{k+1}{j+1}b^{k-j}a^{left( jright) }
label{darij1.eq.thm.1.claim}
tag{1}
end{equation}

for each nonnegative integer $k$.




Proof of Theorem 1. We shall prove eqref{darij1.eq.thm.1.claim} by
induction on $k$:



Induction base: Comparing
begin{equation}
sum_{i=0}^0 b^i ab^{0-i}=underbrace{b^0 }_{=1}aunderbrace{b^{0-0}
}_{=b^0 =1}=a
end{equation}

with
begin{equation}
sum_{j=0}^0 dbinom{0+1}{j+1}b^{0-j}a^{left( jright) }
=underbrace{dbinom{0+1}{0+1}}_{=1}underbrace{b^{0-0}}_{=b^0
=1}underbrace{a^{left( 0right) }}_{=a}=a,
end{equation}

we obtain $sumlimits_{i=0}^0 b^i ab^{0-i}=sumlimits_{j=0}^0 dbinom{0+1}{j+1}
b^{0-j}a^{left( jright) }$
. In other words, eqref{darij1.eq.thm.1.claim}
holds for $k=0$. This completes the induction base.



Induction step: Let $K$ be a positive integer. Assume that
eqref{darij1.eq.thm.1.claim} holds for $k=K-1$. We must prove that
eqref{darij1.eq.thm.1.claim} holds for $k=K$.



We have assumed that eqref{darij1.eq.thm.1.claim} holds for $k=K-1$. In other
words,
begin{align}
sum_{i=0}^{K-1}b^i ab^{left( K-1right) -i} & =sum_{j=0}^{K-1}
dbinom{left( K-1right) +1}{j+1}b^{left( K-1right) -j}a^{left(
jright) }\
& =sum_{j=0}^{K-1}dbinom{K}{j+1}b^{left( K-1right) -j}a^{left(
jright) }
label{darij1.pf.thm.1.2}
tag{2}
end{align}

(since $left( K-1right) +1=K$).



For every nonnegative integer $j$, we have
begin{align*}
a^{left( j+1right) } & =left[ a^{left( jright) },bright]
qquadleft( text{by the recursive definition of }left( a^{left(
0right) },a^{left( 1right) },a^{left( 2right) },ldotsright)
right) \
& =a^{left( jright) }b-ba^{left( jright) }
end{align*}

(by the definition of $left[ a^{left( jright) },bright] $) and thus
begin{equation}
a^{left( jright) }b=ba^{left( jright) }+a^{left( j+1right)
}.
label{darij1.pf.thm.1.3}
tag{3}
end{equation}



Now, we can split off the addend for $i=K$ from the sum $sum_{i=0}^{K}
b^i ab^{K-i}$
. We thus obtain
begin{equation}
sum_{i=0}^{K}b^i ab^{K-i}=sum_{i=0}^{K-1}b^i aunderbrace{b^{K-i}
}_{substack{=b^{left( K-iright) -1}b\text{(since }K-igeq
1\text{(because }ileq K-1text{))}}}+b^{K}aunderbrace{b^{K-K}}_{=b^0
=1}=sum_{i=0}^{K-1}b^i ab^{left( K-iright) -1}b+b^{K}a.
end{equation}

In view of
begin{align*}
& sum_{i=0}^{K-1}b^i aunderbrace{b^{left( K-iright) -1}}
_{substack{=b^{left( K-1right) -i}\text{(since }left( K-iright)
-1=left( K-1right) -itext{)}}}b\
& =sum_{i=0}^{K-1}b^i ab^{left( K-1right) -i}b=left( sum_{i=0}
^{K-1}b^i ab^{left( K-1right) -i}right) b=left( sum_{j=0}
^{K-1}dbinom{K}{j+1}b^{left( K-1right) -j}a^{left( jright) }right)
b\
& qquadleft(
begin{array}
[c]{c}
text{this follows by multiplying both sides of}\
text{the equality eqref{darij1.pf.thm.1.2} by }b
end{array}
right) \
& =sum_{j=0}^{K-1}dbinom{K}{j+1}b^{left( K-1right) -j}
underbrace{a^{left( jright) }b}_{substack{=ba^{left( jright)
}+a^{left( j+1right) }\text{(by eqref{darij1.pf.thm.1.3})}}}=sum
_{j=0}^{K-1}dbinom{K}{j+1}b^{left( K-1right) -j}left( ba^{left(
jright) }+a^{left( j+1right) }right) \
& =sum_{j=0}^{K-1}dbinom{K}{j+1}underbrace{b^{left( K-1right) -j}
b}_{substack{=b^{left( left( K-1right) -jright) +1}=b^{K-j}
\text{(since }left( left( K-1right) -jright) +1=K-jtext{)}
}}a^{left( jright) }+sum_{j=0}^{K-1}dbinom{K}{j+1}underbrace{b^{left(
K-1right) -j}}_{substack{=b^{K-left( j+1right) }\text{(since }left(
K-1right) -j=K-left( j+1right) text{)}}}a^{left( j+1right) }\
& =sum_{j=0}^{K-1}dbinom{K}{j+1}b^{K-j}a^{left( jright) }
+underbrace{sum_{j=0}^{K-1}dbinom{K}{j+1}b^{K-left( j+1right)
}a^{left( j+1right) }}_{substack{=sum_{j=1}^{K}dbinom{K}{j}
b^{K-j}a^{left( jright) }\text{(here, we substituted }jtext{ for
}j+1text{ in the sum)}}}\
& =sum_{j=0}^{K-1}dbinom{K}{j+1}b^{K-j}a^{left( jright) }+sum_{j=1}
^{K}dbinom{K}{j}b^{K-j}a^{left( jright) },
end{align*}

this rewrites as
begin{align}
& sum_{i=0}^{K}b^i ab^{K-i}nonumber\
& =sum_{j=0}^{K-1}dbinom{K}{j+1}b^{K-j}a^{left( jright) }+sum_{j=1}
^{K}dbinom{K}{j}b^{K-j}a^{left( jright) }+b^{K}
a.
label{darij1.pf.thm.1.5}
tag{4}
end{align}



On the other hand, each nonnegative integer $j$ satisfies
begin{equation}
dbinom{K+1}{j+1}=dbinom{K}{j+1}+dbinom{K}{j}
label{darij1.pf.thm.1.7}
tag{5}
end{equation}

(by the recurrence relation of the binomial coefficients). Also, the
nonnegative integers $K$ and $K+1$ satisfy $K+1 > K$; thus,
begin{equation}
dbinom{K}{K+1}=0
label{darij1.pf.thm.1.8}
tag{6}
end{equation}

(because any two nonnegative integers $n$ and $k$ satisfying $k>n$ must
satisfy $dbinom{n}{k}=0$).



Now,
begin{align*}
& sum_{j=0}^{K}underbrace{dbinom{K+1}{j+1}}_{substack{=dbinom{K}
{j+1}+dbinom{K}{j}\text{(by eqref{darij1.pf.thm.1.7})}}}b^{K-j}a^{left(
jright) }\
& =sum_{j=0}^{K}left( dbinom{K}{j+1}+dbinom{K}{j}right) b^{K-j}
a^{left( jright) }\
& =underbrace{sum_{j=0}^{K}dbinom{K}{j+1}b^{K-j}a^{left( jright) }
}_{substack{=sum_{j=0}^{K-1}dbinom{K}{j+1}b^{K-j}a^{left( jright)
}+dbinom{K}{K+1}b^{K-K}a^{left( Kright) }\text{(here, we have split off
the addend for }j=Ktext{ from the sum)}}}\
& qquad+underbrace{sum_{j=0}^{K}dbinom{K}{j}b^{K-j}a^{left( jright) }
}_{substack{=sum_{j=1}^{K}dbinom{K}{j}b^{K-j}a^{left( jright) }
+dbinom{K}{0}b^{K-0}a^{left( 0right) }\text{(here, we have split off
the addend for }j=0text{ from the sum)}}}\
& =sum_{j=0}^{K-1}dbinom{K}{j+1}b^{K-j}a^{left( jright) }
+underbrace{dbinom{K}{K+1}}_{substack{=0\text{(by
eqref{darij1.pf.thm.1.8})}}}b^{K-K}a^{left( Kright) }\
& qquad+sum_{j=1}^{K}dbinom{K}{j}b^{K-j}a^{left( jright) }
+underbrace{dbinom{K}{0}}_{=1}underbrace{b^{K-0}}_{=b^{K}}
underbrace{a^{left( 0right) }}_{=a}\
& =sum_{j=0}^{K-1}dbinom{K}{j+1}b^{K-j}a^{left( jright) }+sum_{j=1}
^{K}dbinom{K}{j}b^{K-j}a^{left( jright) }+b^{K}a.
end{align*}

Comparing this with eqref{darij1.pf.thm.1.5}, we obtain
begin{equation}
sum_{i=0}^{K}b^i ab^{K-i}=sum_{j=0}^{K}dbinom{K+1}{j+1}b^{K-j}a^{left(
jright) }.
end{equation}

In other words, eqref{darij1.eq.thm.1.claim} holds for $k=K$. This completes
the induction step. Thus, eqref{darij1.eq.thm.1.claim} is proven by
induction. Hence, Theorem 1 follows. $blacksquare$



Remark. Theorem 1 also holds if $R$ is a nonunital ring, provided that we interpret all the expressions appearing in eqref{darij1.eq.thm.1.claim} appropriately. (For example, a product of the form "$b^0 a$" has to be interpreted as $a$ even though its sub-expression "$b^0$" is not defined.) The proof we gave above still applies to this situation.






share|cite|improve this answer























    Your Answer





    StackExchange.ifUsing("editor", function () {
    return StackExchange.using("mathjaxEditing", function () {
    StackExchange.MarkdownEditor.creationCallbacks.add(function (editor, postfix) {
    StackExchange.mathjaxEditing.prepareWmdForMathJax(editor, postfix, [["$", "$"], ["\\(","\\)"]]);
    });
    });
    }, "mathjax-editing");

    StackExchange.ready(function() {
    var channelOptions = {
    tags: "".split(" "),
    id: "69"
    };
    initTagRenderer("".split(" "), "".split(" "), channelOptions);

    StackExchange.using("externalEditor", function() {
    // Have to fire editor after snippets, if snippets enabled
    if (StackExchange.settings.snippets.snippetsEnabled) {
    StackExchange.using("snippets", function() {
    createEditor();
    });
    }
    else {
    createEditor();
    }
    });

    function createEditor() {
    StackExchange.prepareEditor({
    heartbeatType: 'answer',
    autoActivateHeartbeat: false,
    convertImagesToLinks: true,
    noModals: true,
    showLowRepImageUploadWarning: true,
    reputationToPostImages: 10,
    bindNavPrevention: true,
    postfix: "",
    imageUploader: {
    brandingHtml: "Powered by u003ca class="icon-imgur-white" href="https://imgur.com/"u003eu003c/au003e",
    contentPolicyHtml: "User contributions licensed under u003ca href="https://creativecommons.org/licenses/by-sa/3.0/"u003ecc by-sa 3.0 with attribution requiredu003c/au003e u003ca href="https://stackoverflow.com/legal/content-policy"u003e(content policy)u003c/au003e",
    allowUrls: true
    },
    noCode: true, onDemand: true,
    discardSelector: ".discard-answer"
    ,immediatelyShowMarkdownHelp:true
    });


    }
    });














    draft saved

    draft discarded


















    StackExchange.ready(
    function () {
    StackExchange.openid.initPostLogin('.new-post-login', 'https%3a%2f%2fmath.stackexchange.com%2fquestions%2f3006004%2fidentity-with-repeatedly-taking-the-commutator-of-a-ring-element%23new-answer', 'question_page');
    }
    );

    Post as a guest















    Required, but never shown

























    1 Answer
    1






    active

    oldest

    votes








    1 Answer
    1






    active

    oldest

    votes









    active

    oldest

    votes






    active

    oldest

    votes









    1














    I outlined two solutions in the comments above; let me expand one of them (the
    inductive one) into full detail in order to have this question answered. Be
    warned: This is going to be a long computation with no twists or surprises.




    Theorem 1. Let $a$ and $b$ be two elements of an (associative, unital,
    noncommutative) ring $R$. For any $xin R$ and $yin R$, we define the
    commutator $left[ x,yright] in R$ of $x$ and $y$ by $left[
    x,yright] =xy-yx$
    . Define a sequence $left( a^{left( 0right)
    },a^{left( 1right) },a^{left( 2right) },ldotsright) $
    of elements
    of $R$ recursively by setting
    begin{align*}
    a^{left( 0right) } & =aqquadtext{and}\
    a^{left( kright) } & =left[ a^{left( k-1right) },bright]
    qquadtext{for each }kgeq 1.
    end{align*}

    Then,
    begin{equation}
    sum_{i=0}^{k} b^i ab^{k-i}
    = sum_{j=0}^{k}dbinom{k+1}{j+1}b^{k-j}a^{left( jright) }
    label{darij1.eq.thm.1.claim}
    tag{1}
    end{equation}

    for each nonnegative integer $k$.




    Proof of Theorem 1. We shall prove eqref{darij1.eq.thm.1.claim} by
    induction on $k$:



    Induction base: Comparing
    begin{equation}
    sum_{i=0}^0 b^i ab^{0-i}=underbrace{b^0 }_{=1}aunderbrace{b^{0-0}
    }_{=b^0 =1}=a
    end{equation}

    with
    begin{equation}
    sum_{j=0}^0 dbinom{0+1}{j+1}b^{0-j}a^{left( jright) }
    =underbrace{dbinom{0+1}{0+1}}_{=1}underbrace{b^{0-0}}_{=b^0
    =1}underbrace{a^{left( 0right) }}_{=a}=a,
    end{equation}

    we obtain $sumlimits_{i=0}^0 b^i ab^{0-i}=sumlimits_{j=0}^0 dbinom{0+1}{j+1}
    b^{0-j}a^{left( jright) }$
    . In other words, eqref{darij1.eq.thm.1.claim}
    holds for $k=0$. This completes the induction base.



    Induction step: Let $K$ be a positive integer. Assume that
    eqref{darij1.eq.thm.1.claim} holds for $k=K-1$. We must prove that
    eqref{darij1.eq.thm.1.claim} holds for $k=K$.



    We have assumed that eqref{darij1.eq.thm.1.claim} holds for $k=K-1$. In other
    words,
    begin{align}
    sum_{i=0}^{K-1}b^i ab^{left( K-1right) -i} & =sum_{j=0}^{K-1}
    dbinom{left( K-1right) +1}{j+1}b^{left( K-1right) -j}a^{left(
    jright) }\
    & =sum_{j=0}^{K-1}dbinom{K}{j+1}b^{left( K-1right) -j}a^{left(
    jright) }
    label{darij1.pf.thm.1.2}
    tag{2}
    end{align}

    (since $left( K-1right) +1=K$).



    For every nonnegative integer $j$, we have
    begin{align*}
    a^{left( j+1right) } & =left[ a^{left( jright) },bright]
    qquadleft( text{by the recursive definition of }left( a^{left(
    0right) },a^{left( 1right) },a^{left( 2right) },ldotsright)
    right) \
    & =a^{left( jright) }b-ba^{left( jright) }
    end{align*}

    (by the definition of $left[ a^{left( jright) },bright] $) and thus
    begin{equation}
    a^{left( jright) }b=ba^{left( jright) }+a^{left( j+1right)
    }.
    label{darij1.pf.thm.1.3}
    tag{3}
    end{equation}



    Now, we can split off the addend for $i=K$ from the sum $sum_{i=0}^{K}
    b^i ab^{K-i}$
    . We thus obtain
    begin{equation}
    sum_{i=0}^{K}b^i ab^{K-i}=sum_{i=0}^{K-1}b^i aunderbrace{b^{K-i}
    }_{substack{=b^{left( K-iright) -1}b\text{(since }K-igeq
    1\text{(because }ileq K-1text{))}}}+b^{K}aunderbrace{b^{K-K}}_{=b^0
    =1}=sum_{i=0}^{K-1}b^i ab^{left( K-iright) -1}b+b^{K}a.
    end{equation}

    In view of
    begin{align*}
    & sum_{i=0}^{K-1}b^i aunderbrace{b^{left( K-iright) -1}}
    _{substack{=b^{left( K-1right) -i}\text{(since }left( K-iright)
    -1=left( K-1right) -itext{)}}}b\
    & =sum_{i=0}^{K-1}b^i ab^{left( K-1right) -i}b=left( sum_{i=0}
    ^{K-1}b^i ab^{left( K-1right) -i}right) b=left( sum_{j=0}
    ^{K-1}dbinom{K}{j+1}b^{left( K-1right) -j}a^{left( jright) }right)
    b\
    & qquadleft(
    begin{array}
    [c]{c}
    text{this follows by multiplying both sides of}\
    text{the equality eqref{darij1.pf.thm.1.2} by }b
    end{array}
    right) \
    & =sum_{j=0}^{K-1}dbinom{K}{j+1}b^{left( K-1right) -j}
    underbrace{a^{left( jright) }b}_{substack{=ba^{left( jright)
    }+a^{left( j+1right) }\text{(by eqref{darij1.pf.thm.1.3})}}}=sum
    _{j=0}^{K-1}dbinom{K}{j+1}b^{left( K-1right) -j}left( ba^{left(
    jright) }+a^{left( j+1right) }right) \
    & =sum_{j=0}^{K-1}dbinom{K}{j+1}underbrace{b^{left( K-1right) -j}
    b}_{substack{=b^{left( left( K-1right) -jright) +1}=b^{K-j}
    \text{(since }left( left( K-1right) -jright) +1=K-jtext{)}
    }}a^{left( jright) }+sum_{j=0}^{K-1}dbinom{K}{j+1}underbrace{b^{left(
    K-1right) -j}}_{substack{=b^{K-left( j+1right) }\text{(since }left(
    K-1right) -j=K-left( j+1right) text{)}}}a^{left( j+1right) }\
    & =sum_{j=0}^{K-1}dbinom{K}{j+1}b^{K-j}a^{left( jright) }
    +underbrace{sum_{j=0}^{K-1}dbinom{K}{j+1}b^{K-left( j+1right)
    }a^{left( j+1right) }}_{substack{=sum_{j=1}^{K}dbinom{K}{j}
    b^{K-j}a^{left( jright) }\text{(here, we substituted }jtext{ for
    }j+1text{ in the sum)}}}\
    & =sum_{j=0}^{K-1}dbinom{K}{j+1}b^{K-j}a^{left( jright) }+sum_{j=1}
    ^{K}dbinom{K}{j}b^{K-j}a^{left( jright) },
    end{align*}

    this rewrites as
    begin{align}
    & sum_{i=0}^{K}b^i ab^{K-i}nonumber\
    & =sum_{j=0}^{K-1}dbinom{K}{j+1}b^{K-j}a^{left( jright) }+sum_{j=1}
    ^{K}dbinom{K}{j}b^{K-j}a^{left( jright) }+b^{K}
    a.
    label{darij1.pf.thm.1.5}
    tag{4}
    end{align}



    On the other hand, each nonnegative integer $j$ satisfies
    begin{equation}
    dbinom{K+1}{j+1}=dbinom{K}{j+1}+dbinom{K}{j}
    label{darij1.pf.thm.1.7}
    tag{5}
    end{equation}

    (by the recurrence relation of the binomial coefficients). Also, the
    nonnegative integers $K$ and $K+1$ satisfy $K+1 > K$; thus,
    begin{equation}
    dbinom{K}{K+1}=0
    label{darij1.pf.thm.1.8}
    tag{6}
    end{equation}

    (because any two nonnegative integers $n$ and $k$ satisfying $k>n$ must
    satisfy $dbinom{n}{k}=0$).



    Now,
    begin{align*}
    & sum_{j=0}^{K}underbrace{dbinom{K+1}{j+1}}_{substack{=dbinom{K}
    {j+1}+dbinom{K}{j}\text{(by eqref{darij1.pf.thm.1.7})}}}b^{K-j}a^{left(
    jright) }\
    & =sum_{j=0}^{K}left( dbinom{K}{j+1}+dbinom{K}{j}right) b^{K-j}
    a^{left( jright) }\
    & =underbrace{sum_{j=0}^{K}dbinom{K}{j+1}b^{K-j}a^{left( jright) }
    }_{substack{=sum_{j=0}^{K-1}dbinom{K}{j+1}b^{K-j}a^{left( jright)
    }+dbinom{K}{K+1}b^{K-K}a^{left( Kright) }\text{(here, we have split off
    the addend for }j=Ktext{ from the sum)}}}\
    & qquad+underbrace{sum_{j=0}^{K}dbinom{K}{j}b^{K-j}a^{left( jright) }
    }_{substack{=sum_{j=1}^{K}dbinom{K}{j}b^{K-j}a^{left( jright) }
    +dbinom{K}{0}b^{K-0}a^{left( 0right) }\text{(here, we have split off
    the addend for }j=0text{ from the sum)}}}\
    & =sum_{j=0}^{K-1}dbinom{K}{j+1}b^{K-j}a^{left( jright) }
    +underbrace{dbinom{K}{K+1}}_{substack{=0\text{(by
    eqref{darij1.pf.thm.1.8})}}}b^{K-K}a^{left( Kright) }\
    & qquad+sum_{j=1}^{K}dbinom{K}{j}b^{K-j}a^{left( jright) }
    +underbrace{dbinom{K}{0}}_{=1}underbrace{b^{K-0}}_{=b^{K}}
    underbrace{a^{left( 0right) }}_{=a}\
    & =sum_{j=0}^{K-1}dbinom{K}{j+1}b^{K-j}a^{left( jright) }+sum_{j=1}
    ^{K}dbinom{K}{j}b^{K-j}a^{left( jright) }+b^{K}a.
    end{align*}

    Comparing this with eqref{darij1.pf.thm.1.5}, we obtain
    begin{equation}
    sum_{i=0}^{K}b^i ab^{K-i}=sum_{j=0}^{K}dbinom{K+1}{j+1}b^{K-j}a^{left(
    jright) }.
    end{equation}

    In other words, eqref{darij1.eq.thm.1.claim} holds for $k=K$. This completes
    the induction step. Thus, eqref{darij1.eq.thm.1.claim} is proven by
    induction. Hence, Theorem 1 follows. $blacksquare$



    Remark. Theorem 1 also holds if $R$ is a nonunital ring, provided that we interpret all the expressions appearing in eqref{darij1.eq.thm.1.claim} appropriately. (For example, a product of the form "$b^0 a$" has to be interpreted as $a$ even though its sub-expression "$b^0$" is not defined.) The proof we gave above still applies to this situation.






    share|cite|improve this answer




























      1














      I outlined two solutions in the comments above; let me expand one of them (the
      inductive one) into full detail in order to have this question answered. Be
      warned: This is going to be a long computation with no twists or surprises.




      Theorem 1. Let $a$ and $b$ be two elements of an (associative, unital,
      noncommutative) ring $R$. For any $xin R$ and $yin R$, we define the
      commutator $left[ x,yright] in R$ of $x$ and $y$ by $left[
      x,yright] =xy-yx$
      . Define a sequence $left( a^{left( 0right)
      },a^{left( 1right) },a^{left( 2right) },ldotsright) $
      of elements
      of $R$ recursively by setting
      begin{align*}
      a^{left( 0right) } & =aqquadtext{and}\
      a^{left( kright) } & =left[ a^{left( k-1right) },bright]
      qquadtext{for each }kgeq 1.
      end{align*}

      Then,
      begin{equation}
      sum_{i=0}^{k} b^i ab^{k-i}
      = sum_{j=0}^{k}dbinom{k+1}{j+1}b^{k-j}a^{left( jright) }
      label{darij1.eq.thm.1.claim}
      tag{1}
      end{equation}

      for each nonnegative integer $k$.




      Proof of Theorem 1. We shall prove eqref{darij1.eq.thm.1.claim} by
      induction on $k$:



      Induction base: Comparing
      begin{equation}
      sum_{i=0}^0 b^i ab^{0-i}=underbrace{b^0 }_{=1}aunderbrace{b^{0-0}
      }_{=b^0 =1}=a
      end{equation}

      with
      begin{equation}
      sum_{j=0}^0 dbinom{0+1}{j+1}b^{0-j}a^{left( jright) }
      =underbrace{dbinom{0+1}{0+1}}_{=1}underbrace{b^{0-0}}_{=b^0
      =1}underbrace{a^{left( 0right) }}_{=a}=a,
      end{equation}

      we obtain $sumlimits_{i=0}^0 b^i ab^{0-i}=sumlimits_{j=0}^0 dbinom{0+1}{j+1}
      b^{0-j}a^{left( jright) }$
      . In other words, eqref{darij1.eq.thm.1.claim}
      holds for $k=0$. This completes the induction base.



      Induction step: Let $K$ be a positive integer. Assume that
      eqref{darij1.eq.thm.1.claim} holds for $k=K-1$. We must prove that
      eqref{darij1.eq.thm.1.claim} holds for $k=K$.



      We have assumed that eqref{darij1.eq.thm.1.claim} holds for $k=K-1$. In other
      words,
      begin{align}
      sum_{i=0}^{K-1}b^i ab^{left( K-1right) -i} & =sum_{j=0}^{K-1}
      dbinom{left( K-1right) +1}{j+1}b^{left( K-1right) -j}a^{left(
      jright) }\
      & =sum_{j=0}^{K-1}dbinom{K}{j+1}b^{left( K-1right) -j}a^{left(
      jright) }
      label{darij1.pf.thm.1.2}
      tag{2}
      end{align}

      (since $left( K-1right) +1=K$).



      For every nonnegative integer $j$, we have
      begin{align*}
      a^{left( j+1right) } & =left[ a^{left( jright) },bright]
      qquadleft( text{by the recursive definition of }left( a^{left(
      0right) },a^{left( 1right) },a^{left( 2right) },ldotsright)
      right) \
      & =a^{left( jright) }b-ba^{left( jright) }
      end{align*}

      (by the definition of $left[ a^{left( jright) },bright] $) and thus
      begin{equation}
      a^{left( jright) }b=ba^{left( jright) }+a^{left( j+1right)
      }.
      label{darij1.pf.thm.1.3}
      tag{3}
      end{equation}



      Now, we can split off the addend for $i=K$ from the sum $sum_{i=0}^{K}
      b^i ab^{K-i}$
      . We thus obtain
      begin{equation}
      sum_{i=0}^{K}b^i ab^{K-i}=sum_{i=0}^{K-1}b^i aunderbrace{b^{K-i}
      }_{substack{=b^{left( K-iright) -1}b\text{(since }K-igeq
      1\text{(because }ileq K-1text{))}}}+b^{K}aunderbrace{b^{K-K}}_{=b^0
      =1}=sum_{i=0}^{K-1}b^i ab^{left( K-iright) -1}b+b^{K}a.
      end{equation}

      In view of
      begin{align*}
      & sum_{i=0}^{K-1}b^i aunderbrace{b^{left( K-iright) -1}}
      _{substack{=b^{left( K-1right) -i}\text{(since }left( K-iright)
      -1=left( K-1right) -itext{)}}}b\
      & =sum_{i=0}^{K-1}b^i ab^{left( K-1right) -i}b=left( sum_{i=0}
      ^{K-1}b^i ab^{left( K-1right) -i}right) b=left( sum_{j=0}
      ^{K-1}dbinom{K}{j+1}b^{left( K-1right) -j}a^{left( jright) }right)
      b\
      & qquadleft(
      begin{array}
      [c]{c}
      text{this follows by multiplying both sides of}\
      text{the equality eqref{darij1.pf.thm.1.2} by }b
      end{array}
      right) \
      & =sum_{j=0}^{K-1}dbinom{K}{j+1}b^{left( K-1right) -j}
      underbrace{a^{left( jright) }b}_{substack{=ba^{left( jright)
      }+a^{left( j+1right) }\text{(by eqref{darij1.pf.thm.1.3})}}}=sum
      _{j=0}^{K-1}dbinom{K}{j+1}b^{left( K-1right) -j}left( ba^{left(
      jright) }+a^{left( j+1right) }right) \
      & =sum_{j=0}^{K-1}dbinom{K}{j+1}underbrace{b^{left( K-1right) -j}
      b}_{substack{=b^{left( left( K-1right) -jright) +1}=b^{K-j}
      \text{(since }left( left( K-1right) -jright) +1=K-jtext{)}
      }}a^{left( jright) }+sum_{j=0}^{K-1}dbinom{K}{j+1}underbrace{b^{left(
      K-1right) -j}}_{substack{=b^{K-left( j+1right) }\text{(since }left(
      K-1right) -j=K-left( j+1right) text{)}}}a^{left( j+1right) }\
      & =sum_{j=0}^{K-1}dbinom{K}{j+1}b^{K-j}a^{left( jright) }
      +underbrace{sum_{j=0}^{K-1}dbinom{K}{j+1}b^{K-left( j+1right)
      }a^{left( j+1right) }}_{substack{=sum_{j=1}^{K}dbinom{K}{j}
      b^{K-j}a^{left( jright) }\text{(here, we substituted }jtext{ for
      }j+1text{ in the sum)}}}\
      & =sum_{j=0}^{K-1}dbinom{K}{j+1}b^{K-j}a^{left( jright) }+sum_{j=1}
      ^{K}dbinom{K}{j}b^{K-j}a^{left( jright) },
      end{align*}

      this rewrites as
      begin{align}
      & sum_{i=0}^{K}b^i ab^{K-i}nonumber\
      & =sum_{j=0}^{K-1}dbinom{K}{j+1}b^{K-j}a^{left( jright) }+sum_{j=1}
      ^{K}dbinom{K}{j}b^{K-j}a^{left( jright) }+b^{K}
      a.
      label{darij1.pf.thm.1.5}
      tag{4}
      end{align}



      On the other hand, each nonnegative integer $j$ satisfies
      begin{equation}
      dbinom{K+1}{j+1}=dbinom{K}{j+1}+dbinom{K}{j}
      label{darij1.pf.thm.1.7}
      tag{5}
      end{equation}

      (by the recurrence relation of the binomial coefficients). Also, the
      nonnegative integers $K$ and $K+1$ satisfy $K+1 > K$; thus,
      begin{equation}
      dbinom{K}{K+1}=0
      label{darij1.pf.thm.1.8}
      tag{6}
      end{equation}

      (because any two nonnegative integers $n$ and $k$ satisfying $k>n$ must
      satisfy $dbinom{n}{k}=0$).



      Now,
      begin{align*}
      & sum_{j=0}^{K}underbrace{dbinom{K+1}{j+1}}_{substack{=dbinom{K}
      {j+1}+dbinom{K}{j}\text{(by eqref{darij1.pf.thm.1.7})}}}b^{K-j}a^{left(
      jright) }\
      & =sum_{j=0}^{K}left( dbinom{K}{j+1}+dbinom{K}{j}right) b^{K-j}
      a^{left( jright) }\
      & =underbrace{sum_{j=0}^{K}dbinom{K}{j+1}b^{K-j}a^{left( jright) }
      }_{substack{=sum_{j=0}^{K-1}dbinom{K}{j+1}b^{K-j}a^{left( jright)
      }+dbinom{K}{K+1}b^{K-K}a^{left( Kright) }\text{(here, we have split off
      the addend for }j=Ktext{ from the sum)}}}\
      & qquad+underbrace{sum_{j=0}^{K}dbinom{K}{j}b^{K-j}a^{left( jright) }
      }_{substack{=sum_{j=1}^{K}dbinom{K}{j}b^{K-j}a^{left( jright) }
      +dbinom{K}{0}b^{K-0}a^{left( 0right) }\text{(here, we have split off
      the addend for }j=0text{ from the sum)}}}\
      & =sum_{j=0}^{K-1}dbinom{K}{j+1}b^{K-j}a^{left( jright) }
      +underbrace{dbinom{K}{K+1}}_{substack{=0\text{(by
      eqref{darij1.pf.thm.1.8})}}}b^{K-K}a^{left( Kright) }\
      & qquad+sum_{j=1}^{K}dbinom{K}{j}b^{K-j}a^{left( jright) }
      +underbrace{dbinom{K}{0}}_{=1}underbrace{b^{K-0}}_{=b^{K}}
      underbrace{a^{left( 0right) }}_{=a}\
      & =sum_{j=0}^{K-1}dbinom{K}{j+1}b^{K-j}a^{left( jright) }+sum_{j=1}
      ^{K}dbinom{K}{j}b^{K-j}a^{left( jright) }+b^{K}a.
      end{align*}

      Comparing this with eqref{darij1.pf.thm.1.5}, we obtain
      begin{equation}
      sum_{i=0}^{K}b^i ab^{K-i}=sum_{j=0}^{K}dbinom{K+1}{j+1}b^{K-j}a^{left(
      jright) }.
      end{equation}

      In other words, eqref{darij1.eq.thm.1.claim} holds for $k=K$. This completes
      the induction step. Thus, eqref{darij1.eq.thm.1.claim} is proven by
      induction. Hence, Theorem 1 follows. $blacksquare$



      Remark. Theorem 1 also holds if $R$ is a nonunital ring, provided that we interpret all the expressions appearing in eqref{darij1.eq.thm.1.claim} appropriately. (For example, a product of the form "$b^0 a$" has to be interpreted as $a$ even though its sub-expression "$b^0$" is not defined.) The proof we gave above still applies to this situation.






      share|cite|improve this answer


























        1












        1








        1






        I outlined two solutions in the comments above; let me expand one of them (the
        inductive one) into full detail in order to have this question answered. Be
        warned: This is going to be a long computation with no twists or surprises.




        Theorem 1. Let $a$ and $b$ be two elements of an (associative, unital,
        noncommutative) ring $R$. For any $xin R$ and $yin R$, we define the
        commutator $left[ x,yright] in R$ of $x$ and $y$ by $left[
        x,yright] =xy-yx$
        . Define a sequence $left( a^{left( 0right)
        },a^{left( 1right) },a^{left( 2right) },ldotsright) $
        of elements
        of $R$ recursively by setting
        begin{align*}
        a^{left( 0right) } & =aqquadtext{and}\
        a^{left( kright) } & =left[ a^{left( k-1right) },bright]
        qquadtext{for each }kgeq 1.
        end{align*}

        Then,
        begin{equation}
        sum_{i=0}^{k} b^i ab^{k-i}
        = sum_{j=0}^{k}dbinom{k+1}{j+1}b^{k-j}a^{left( jright) }
        label{darij1.eq.thm.1.claim}
        tag{1}
        end{equation}

        for each nonnegative integer $k$.




        Proof of Theorem 1. We shall prove eqref{darij1.eq.thm.1.claim} by
        induction on $k$:



        Induction base: Comparing
        begin{equation}
        sum_{i=0}^0 b^i ab^{0-i}=underbrace{b^0 }_{=1}aunderbrace{b^{0-0}
        }_{=b^0 =1}=a
        end{equation}

        with
        begin{equation}
        sum_{j=0}^0 dbinom{0+1}{j+1}b^{0-j}a^{left( jright) }
        =underbrace{dbinom{0+1}{0+1}}_{=1}underbrace{b^{0-0}}_{=b^0
        =1}underbrace{a^{left( 0right) }}_{=a}=a,
        end{equation}

        we obtain $sumlimits_{i=0}^0 b^i ab^{0-i}=sumlimits_{j=0}^0 dbinom{0+1}{j+1}
        b^{0-j}a^{left( jright) }$
        . In other words, eqref{darij1.eq.thm.1.claim}
        holds for $k=0$. This completes the induction base.



        Induction step: Let $K$ be a positive integer. Assume that
        eqref{darij1.eq.thm.1.claim} holds for $k=K-1$. We must prove that
        eqref{darij1.eq.thm.1.claim} holds for $k=K$.



        We have assumed that eqref{darij1.eq.thm.1.claim} holds for $k=K-1$. In other
        words,
        begin{align}
        sum_{i=0}^{K-1}b^i ab^{left( K-1right) -i} & =sum_{j=0}^{K-1}
        dbinom{left( K-1right) +1}{j+1}b^{left( K-1right) -j}a^{left(
        jright) }\
        & =sum_{j=0}^{K-1}dbinom{K}{j+1}b^{left( K-1right) -j}a^{left(
        jright) }
        label{darij1.pf.thm.1.2}
        tag{2}
        end{align}

        (since $left( K-1right) +1=K$).



        For every nonnegative integer $j$, we have
        begin{align*}
        a^{left( j+1right) } & =left[ a^{left( jright) },bright]
        qquadleft( text{by the recursive definition of }left( a^{left(
        0right) },a^{left( 1right) },a^{left( 2right) },ldotsright)
        right) \
        & =a^{left( jright) }b-ba^{left( jright) }
        end{align*}

        (by the definition of $left[ a^{left( jright) },bright] $) and thus
        begin{equation}
        a^{left( jright) }b=ba^{left( jright) }+a^{left( j+1right)
        }.
        label{darij1.pf.thm.1.3}
        tag{3}
        end{equation}



        Now, we can split off the addend for $i=K$ from the sum $sum_{i=0}^{K}
        b^i ab^{K-i}$
        . We thus obtain
        begin{equation}
        sum_{i=0}^{K}b^i ab^{K-i}=sum_{i=0}^{K-1}b^i aunderbrace{b^{K-i}
        }_{substack{=b^{left( K-iright) -1}b\text{(since }K-igeq
        1\text{(because }ileq K-1text{))}}}+b^{K}aunderbrace{b^{K-K}}_{=b^0
        =1}=sum_{i=0}^{K-1}b^i ab^{left( K-iright) -1}b+b^{K}a.
        end{equation}

        In view of
        begin{align*}
        & sum_{i=0}^{K-1}b^i aunderbrace{b^{left( K-iright) -1}}
        _{substack{=b^{left( K-1right) -i}\text{(since }left( K-iright)
        -1=left( K-1right) -itext{)}}}b\
        & =sum_{i=0}^{K-1}b^i ab^{left( K-1right) -i}b=left( sum_{i=0}
        ^{K-1}b^i ab^{left( K-1right) -i}right) b=left( sum_{j=0}
        ^{K-1}dbinom{K}{j+1}b^{left( K-1right) -j}a^{left( jright) }right)
        b\
        & qquadleft(
        begin{array}
        [c]{c}
        text{this follows by multiplying both sides of}\
        text{the equality eqref{darij1.pf.thm.1.2} by }b
        end{array}
        right) \
        & =sum_{j=0}^{K-1}dbinom{K}{j+1}b^{left( K-1right) -j}
        underbrace{a^{left( jright) }b}_{substack{=ba^{left( jright)
        }+a^{left( j+1right) }\text{(by eqref{darij1.pf.thm.1.3})}}}=sum
        _{j=0}^{K-1}dbinom{K}{j+1}b^{left( K-1right) -j}left( ba^{left(
        jright) }+a^{left( j+1right) }right) \
        & =sum_{j=0}^{K-1}dbinom{K}{j+1}underbrace{b^{left( K-1right) -j}
        b}_{substack{=b^{left( left( K-1right) -jright) +1}=b^{K-j}
        \text{(since }left( left( K-1right) -jright) +1=K-jtext{)}
        }}a^{left( jright) }+sum_{j=0}^{K-1}dbinom{K}{j+1}underbrace{b^{left(
        K-1right) -j}}_{substack{=b^{K-left( j+1right) }\text{(since }left(
        K-1right) -j=K-left( j+1right) text{)}}}a^{left( j+1right) }\
        & =sum_{j=0}^{K-1}dbinom{K}{j+1}b^{K-j}a^{left( jright) }
        +underbrace{sum_{j=0}^{K-1}dbinom{K}{j+1}b^{K-left( j+1right)
        }a^{left( j+1right) }}_{substack{=sum_{j=1}^{K}dbinom{K}{j}
        b^{K-j}a^{left( jright) }\text{(here, we substituted }jtext{ for
        }j+1text{ in the sum)}}}\
        & =sum_{j=0}^{K-1}dbinom{K}{j+1}b^{K-j}a^{left( jright) }+sum_{j=1}
        ^{K}dbinom{K}{j}b^{K-j}a^{left( jright) },
        end{align*}

        this rewrites as
        begin{align}
        & sum_{i=0}^{K}b^i ab^{K-i}nonumber\
        & =sum_{j=0}^{K-1}dbinom{K}{j+1}b^{K-j}a^{left( jright) }+sum_{j=1}
        ^{K}dbinom{K}{j}b^{K-j}a^{left( jright) }+b^{K}
        a.
        label{darij1.pf.thm.1.5}
        tag{4}
        end{align}



        On the other hand, each nonnegative integer $j$ satisfies
        begin{equation}
        dbinom{K+1}{j+1}=dbinom{K}{j+1}+dbinom{K}{j}
        label{darij1.pf.thm.1.7}
        tag{5}
        end{equation}

        (by the recurrence relation of the binomial coefficients). Also, the
        nonnegative integers $K$ and $K+1$ satisfy $K+1 > K$; thus,
        begin{equation}
        dbinom{K}{K+1}=0
        label{darij1.pf.thm.1.8}
        tag{6}
        end{equation}

        (because any two nonnegative integers $n$ and $k$ satisfying $k>n$ must
        satisfy $dbinom{n}{k}=0$).



        Now,
        begin{align*}
        & sum_{j=0}^{K}underbrace{dbinom{K+1}{j+1}}_{substack{=dbinom{K}
        {j+1}+dbinom{K}{j}\text{(by eqref{darij1.pf.thm.1.7})}}}b^{K-j}a^{left(
        jright) }\
        & =sum_{j=0}^{K}left( dbinom{K}{j+1}+dbinom{K}{j}right) b^{K-j}
        a^{left( jright) }\
        & =underbrace{sum_{j=0}^{K}dbinom{K}{j+1}b^{K-j}a^{left( jright) }
        }_{substack{=sum_{j=0}^{K-1}dbinom{K}{j+1}b^{K-j}a^{left( jright)
        }+dbinom{K}{K+1}b^{K-K}a^{left( Kright) }\text{(here, we have split off
        the addend for }j=Ktext{ from the sum)}}}\
        & qquad+underbrace{sum_{j=0}^{K}dbinom{K}{j}b^{K-j}a^{left( jright) }
        }_{substack{=sum_{j=1}^{K}dbinom{K}{j}b^{K-j}a^{left( jright) }
        +dbinom{K}{0}b^{K-0}a^{left( 0right) }\text{(here, we have split off
        the addend for }j=0text{ from the sum)}}}\
        & =sum_{j=0}^{K-1}dbinom{K}{j+1}b^{K-j}a^{left( jright) }
        +underbrace{dbinom{K}{K+1}}_{substack{=0\text{(by
        eqref{darij1.pf.thm.1.8})}}}b^{K-K}a^{left( Kright) }\
        & qquad+sum_{j=1}^{K}dbinom{K}{j}b^{K-j}a^{left( jright) }
        +underbrace{dbinom{K}{0}}_{=1}underbrace{b^{K-0}}_{=b^{K}}
        underbrace{a^{left( 0right) }}_{=a}\
        & =sum_{j=0}^{K-1}dbinom{K}{j+1}b^{K-j}a^{left( jright) }+sum_{j=1}
        ^{K}dbinom{K}{j}b^{K-j}a^{left( jright) }+b^{K}a.
        end{align*}

        Comparing this with eqref{darij1.pf.thm.1.5}, we obtain
        begin{equation}
        sum_{i=0}^{K}b^i ab^{K-i}=sum_{j=0}^{K}dbinom{K+1}{j+1}b^{K-j}a^{left(
        jright) }.
        end{equation}

        In other words, eqref{darij1.eq.thm.1.claim} holds for $k=K$. This completes
        the induction step. Thus, eqref{darij1.eq.thm.1.claim} is proven by
        induction. Hence, Theorem 1 follows. $blacksquare$



        Remark. Theorem 1 also holds if $R$ is a nonunital ring, provided that we interpret all the expressions appearing in eqref{darij1.eq.thm.1.claim} appropriately. (For example, a product of the form "$b^0 a$" has to be interpreted as $a$ even though its sub-expression "$b^0$" is not defined.) The proof we gave above still applies to this situation.






        share|cite|improve this answer














        I outlined two solutions in the comments above; let me expand one of them (the
        inductive one) into full detail in order to have this question answered. Be
        warned: This is going to be a long computation with no twists or surprises.




        Theorem 1. Let $a$ and $b$ be two elements of an (associative, unital,
        noncommutative) ring $R$. For any $xin R$ and $yin R$, we define the
        commutator $left[ x,yright] in R$ of $x$ and $y$ by $left[
        x,yright] =xy-yx$
        . Define a sequence $left( a^{left( 0right)
        },a^{left( 1right) },a^{left( 2right) },ldotsright) $
        of elements
        of $R$ recursively by setting
        begin{align*}
        a^{left( 0right) } & =aqquadtext{and}\
        a^{left( kright) } & =left[ a^{left( k-1right) },bright]
        qquadtext{for each }kgeq 1.
        end{align*}

        Then,
        begin{equation}
        sum_{i=0}^{k} b^i ab^{k-i}
        = sum_{j=0}^{k}dbinom{k+1}{j+1}b^{k-j}a^{left( jright) }
        label{darij1.eq.thm.1.claim}
        tag{1}
        end{equation}

        for each nonnegative integer $k$.




        Proof of Theorem 1. We shall prove eqref{darij1.eq.thm.1.claim} by
        induction on $k$:



        Induction base: Comparing
        begin{equation}
        sum_{i=0}^0 b^i ab^{0-i}=underbrace{b^0 }_{=1}aunderbrace{b^{0-0}
        }_{=b^0 =1}=a
        end{equation}

        with
        begin{equation}
        sum_{j=0}^0 dbinom{0+1}{j+1}b^{0-j}a^{left( jright) }
        =underbrace{dbinom{0+1}{0+1}}_{=1}underbrace{b^{0-0}}_{=b^0
        =1}underbrace{a^{left( 0right) }}_{=a}=a,
        end{equation}

        we obtain $sumlimits_{i=0}^0 b^i ab^{0-i}=sumlimits_{j=0}^0 dbinom{0+1}{j+1}
        b^{0-j}a^{left( jright) }$
        . In other words, eqref{darij1.eq.thm.1.claim}
        holds for $k=0$. This completes the induction base.



        Induction step: Let $K$ be a positive integer. Assume that
        eqref{darij1.eq.thm.1.claim} holds for $k=K-1$. We must prove that
        eqref{darij1.eq.thm.1.claim} holds for $k=K$.



        We have assumed that eqref{darij1.eq.thm.1.claim} holds for $k=K-1$. In other
        words,
        begin{align}
        sum_{i=0}^{K-1}b^i ab^{left( K-1right) -i} & =sum_{j=0}^{K-1}
        dbinom{left( K-1right) +1}{j+1}b^{left( K-1right) -j}a^{left(
        jright) }\
        & =sum_{j=0}^{K-1}dbinom{K}{j+1}b^{left( K-1right) -j}a^{left(
        jright) }
        label{darij1.pf.thm.1.2}
        tag{2}
        end{align}

        (since $left( K-1right) +1=K$).



        For every nonnegative integer $j$, we have
        begin{align*}
        a^{left( j+1right) } & =left[ a^{left( jright) },bright]
        qquadleft( text{by the recursive definition of }left( a^{left(
        0right) },a^{left( 1right) },a^{left( 2right) },ldotsright)
        right) \
        & =a^{left( jright) }b-ba^{left( jright) }
        end{align*}

        (by the definition of $left[ a^{left( jright) },bright] $) and thus
        begin{equation}
        a^{left( jright) }b=ba^{left( jright) }+a^{left( j+1right)
        }.
        label{darij1.pf.thm.1.3}
        tag{3}
        end{equation}



        Now, we can split off the addend for $i=K$ from the sum $sum_{i=0}^{K}
        b^i ab^{K-i}$
        . We thus obtain
        begin{equation}
        sum_{i=0}^{K}b^i ab^{K-i}=sum_{i=0}^{K-1}b^i aunderbrace{b^{K-i}
        }_{substack{=b^{left( K-iright) -1}b\text{(since }K-igeq
        1\text{(because }ileq K-1text{))}}}+b^{K}aunderbrace{b^{K-K}}_{=b^0
        =1}=sum_{i=0}^{K-1}b^i ab^{left( K-iright) -1}b+b^{K}a.
        end{equation}

        In view of
        begin{align*}
        & sum_{i=0}^{K-1}b^i aunderbrace{b^{left( K-iright) -1}}
        _{substack{=b^{left( K-1right) -i}\text{(since }left( K-iright)
        -1=left( K-1right) -itext{)}}}b\
        & =sum_{i=0}^{K-1}b^i ab^{left( K-1right) -i}b=left( sum_{i=0}
        ^{K-1}b^i ab^{left( K-1right) -i}right) b=left( sum_{j=0}
        ^{K-1}dbinom{K}{j+1}b^{left( K-1right) -j}a^{left( jright) }right)
        b\
        & qquadleft(
        begin{array}
        [c]{c}
        text{this follows by multiplying both sides of}\
        text{the equality eqref{darij1.pf.thm.1.2} by }b
        end{array}
        right) \
        & =sum_{j=0}^{K-1}dbinom{K}{j+1}b^{left( K-1right) -j}
        underbrace{a^{left( jright) }b}_{substack{=ba^{left( jright)
        }+a^{left( j+1right) }\text{(by eqref{darij1.pf.thm.1.3})}}}=sum
        _{j=0}^{K-1}dbinom{K}{j+1}b^{left( K-1right) -j}left( ba^{left(
        jright) }+a^{left( j+1right) }right) \
        & =sum_{j=0}^{K-1}dbinom{K}{j+1}underbrace{b^{left( K-1right) -j}
        b}_{substack{=b^{left( left( K-1right) -jright) +1}=b^{K-j}
        \text{(since }left( left( K-1right) -jright) +1=K-jtext{)}
        }}a^{left( jright) }+sum_{j=0}^{K-1}dbinom{K}{j+1}underbrace{b^{left(
        K-1right) -j}}_{substack{=b^{K-left( j+1right) }\text{(since }left(
        K-1right) -j=K-left( j+1right) text{)}}}a^{left( j+1right) }\
        & =sum_{j=0}^{K-1}dbinom{K}{j+1}b^{K-j}a^{left( jright) }
        +underbrace{sum_{j=0}^{K-1}dbinom{K}{j+1}b^{K-left( j+1right)
        }a^{left( j+1right) }}_{substack{=sum_{j=1}^{K}dbinom{K}{j}
        b^{K-j}a^{left( jright) }\text{(here, we substituted }jtext{ for
        }j+1text{ in the sum)}}}\
        & =sum_{j=0}^{K-1}dbinom{K}{j+1}b^{K-j}a^{left( jright) }+sum_{j=1}
        ^{K}dbinom{K}{j}b^{K-j}a^{left( jright) },
        end{align*}

        this rewrites as
        begin{align}
        & sum_{i=0}^{K}b^i ab^{K-i}nonumber\
        & =sum_{j=0}^{K-1}dbinom{K}{j+1}b^{K-j}a^{left( jright) }+sum_{j=1}
        ^{K}dbinom{K}{j}b^{K-j}a^{left( jright) }+b^{K}
        a.
        label{darij1.pf.thm.1.5}
        tag{4}
        end{align}



        On the other hand, each nonnegative integer $j$ satisfies
        begin{equation}
        dbinom{K+1}{j+1}=dbinom{K}{j+1}+dbinom{K}{j}
        label{darij1.pf.thm.1.7}
        tag{5}
        end{equation}

        (by the recurrence relation of the binomial coefficients). Also, the
        nonnegative integers $K$ and $K+1$ satisfy $K+1 > K$; thus,
        begin{equation}
        dbinom{K}{K+1}=0
        label{darij1.pf.thm.1.8}
        tag{6}
        end{equation}

        (because any two nonnegative integers $n$ and $k$ satisfying $k>n$ must
        satisfy $dbinom{n}{k}=0$).



        Now,
        begin{align*}
        & sum_{j=0}^{K}underbrace{dbinom{K+1}{j+1}}_{substack{=dbinom{K}
        {j+1}+dbinom{K}{j}\text{(by eqref{darij1.pf.thm.1.7})}}}b^{K-j}a^{left(
        jright) }\
        & =sum_{j=0}^{K}left( dbinom{K}{j+1}+dbinom{K}{j}right) b^{K-j}
        a^{left( jright) }\
        & =underbrace{sum_{j=0}^{K}dbinom{K}{j+1}b^{K-j}a^{left( jright) }
        }_{substack{=sum_{j=0}^{K-1}dbinom{K}{j+1}b^{K-j}a^{left( jright)
        }+dbinom{K}{K+1}b^{K-K}a^{left( Kright) }\text{(here, we have split off
        the addend for }j=Ktext{ from the sum)}}}\
        & qquad+underbrace{sum_{j=0}^{K}dbinom{K}{j}b^{K-j}a^{left( jright) }
        }_{substack{=sum_{j=1}^{K}dbinom{K}{j}b^{K-j}a^{left( jright) }
        +dbinom{K}{0}b^{K-0}a^{left( 0right) }\text{(here, we have split off
        the addend for }j=0text{ from the sum)}}}\
        & =sum_{j=0}^{K-1}dbinom{K}{j+1}b^{K-j}a^{left( jright) }
        +underbrace{dbinom{K}{K+1}}_{substack{=0\text{(by
        eqref{darij1.pf.thm.1.8})}}}b^{K-K}a^{left( Kright) }\
        & qquad+sum_{j=1}^{K}dbinom{K}{j}b^{K-j}a^{left( jright) }
        +underbrace{dbinom{K}{0}}_{=1}underbrace{b^{K-0}}_{=b^{K}}
        underbrace{a^{left( 0right) }}_{=a}\
        & =sum_{j=0}^{K-1}dbinom{K}{j+1}b^{K-j}a^{left( jright) }+sum_{j=1}
        ^{K}dbinom{K}{j}b^{K-j}a^{left( jright) }+b^{K}a.
        end{align*}

        Comparing this with eqref{darij1.pf.thm.1.5}, we obtain
        begin{equation}
        sum_{i=0}^{K}b^i ab^{K-i}=sum_{j=0}^{K}dbinom{K+1}{j+1}b^{K-j}a^{left(
        jright) }.
        end{equation}

        In other words, eqref{darij1.eq.thm.1.claim} holds for $k=K$. This completes
        the induction step. Thus, eqref{darij1.eq.thm.1.claim} is proven by
        induction. Hence, Theorem 1 follows. $blacksquare$



        Remark. Theorem 1 also holds if $R$ is a nonunital ring, provided that we interpret all the expressions appearing in eqref{darij1.eq.thm.1.claim} appropriately. (For example, a product of the form "$b^0 a$" has to be interpreted as $a$ even though its sub-expression "$b^0$" is not defined.) The proof we gave above still applies to this situation.







        share|cite|improve this answer














        share|cite|improve this answer



        share|cite|improve this answer








        edited yesterday

























        answered Nov 26 '18 at 3:00









        darij grinberg

        10.2k33062




        10.2k33062






























            draft saved

            draft discarded




















































            Thanks for contributing an answer to Mathematics Stack Exchange!


            • Please be sure to answer the question. Provide details and share your research!

            But avoid



            • Asking for help, clarification, or responding to other answers.

            • Making statements based on opinion; back them up with references or personal experience.


            Use MathJax to format equations. MathJax reference.


            To learn more, see our tips on writing great answers.





            Some of your past answers have not been well-received, and you're in danger of being blocked from answering.


            Please pay close attention to the following guidance:


            • Please be sure to answer the question. Provide details and share your research!

            But avoid



            • Asking for help, clarification, or responding to other answers.

            • Making statements based on opinion; back them up with references or personal experience.


            To learn more, see our tips on writing great answers.




            draft saved


            draft discarded














            StackExchange.ready(
            function () {
            StackExchange.openid.initPostLogin('.new-post-login', 'https%3a%2f%2fmath.stackexchange.com%2fquestions%2f3006004%2fidentity-with-repeatedly-taking-the-commutator-of-a-ring-element%23new-answer', 'question_page');
            }
            );

            Post as a guest















            Required, but never shown





















































            Required, but never shown














            Required, but never shown












            Required, but never shown







            Required, but never shown

































            Required, but never shown














            Required, but never shown












            Required, but never shown







            Required, but never shown







            Popular posts from this blog

            Mario Kart Wii

            What does “Dominus providebit” mean?

            Antonio Litta Visconti Arese